Difference between revisions of "2022 AMC 12B Problems/Problem 19"

(Problem)
(Solution 3 (Law of Cosine))
 
(14 intermediate revisions by 7 users not shown)
Line 1: Line 1:
 
== Problem ==
 
== Problem ==
  
In <math>\triangle ABC</math> medians <math>\overline{\rm AD}</math> and <math>\overline{\rm BE}</math> intersect at <math>G</math> and <math>\triangle AGE</math> is equilateral. Then <math>\cos(C)</math> can be written as <math>\frac{m\sqrt{p}}{n}</math>, where <math>m</math> and <math>n</math> are relatively prime positive integers and <math>p</math> is a positive integer not divisible by the square of any prime. What is <math>m+n+p</math>?
+
In <math>\triangle{ABC}</math> medians <math>\overline{AD}</math> and <math>\overline{BE}</math> intersect at <math>G</math> and <math>\triangle{AGE}</math> is equilateral. Then <math>\cos(C)</math> can be written as <math>\frac{m\sqrt p}n</math>, where <math>m</math> and <math>n</math> are relatively prime positive integers and <math>p</math> is a positive integer not divisible by the square of any prime. What is <math>m+n+p?</math>
  
<math>\textbf{(A)}\ 44 \qquad
+
<math>\textbf{(A) }44 \qquad \textbf{(B) }48 \qquad \textbf{(C) }52 \qquad \textbf{(D) }56 \qquad \textbf{(E) }60</math>
\textbf{(B)}\ 48 \qquad
 
\textbf{(C)}\ 52 \qquad
 
\textbf{(D)}\ 56 \qquad
 
\textbf{(E)}\ 60 \qquad</math>
 
  
==Solution 1: Law of Cosines==
+
== Diagram ==
Note: can someone add the diagram here please, I don't know how to do that
+
<asy>
 +
            import geometry;
 +
            unitsize(2cm);
  
Let <math>\overline{AG}=\overline{AE}=\overline{EG}=2x</math>. Since <math>E</math> is the midpoint of <math>\overline{AC}</math>, <math>\overline{EC}</math> must also be <math>2x</math>.
+
real arg(pair p) {
 +
              return atan2(p.y, p.x) * 180/pi;
 +
            }
  
Since the centroid splits the median in a <math>2:1</math> ratio, <math>\overline{GD}</math> must be equal to <math>x</math> and <math>\overline{BG}</math> must be equal to <math>4x</math>.
+
            pair G=(0,0),E=(1,0),A=(1/2,sqrt(3)/2),D=1.5*G-0.5*A,C=2*E-A,B=2*D-C;
  
Applying Law of Cosines on <math>\triangle{}ADC</math> and <math>\triangle{}AGB</math> yields <math>\overline{AB}=\sqrt{28}x</math> and <math>\overline{CD}=\overline{BD}=\sqrt{13}x</math>. Finally, applying Law of Cosines on <math>\triangle{}ABC</math> yields <math>\cos{C}=\frac{5}{2\sqrt{13}}=\frac{5\sqrt{13}}{26}</math>. The requested sum is <math>5+13+26=44</math>.
+
            pair t(pair p) {
 +
                return rotate(-arg(dir(B--C)))*p;
 +
            }
 +
 
 +
 
 +
            path t(path p) {
 +
                return rotate(-arg(dir(B--C)))*p;
 +
            }
 +
 
 +
            void d(path p, pen q = black+linewidth(1.5)) {
 +
                draw(t(p),q);
 +
            }
 +
 
 +
            void o(pair p, pen q = 5+black) {
 +
                dot(t(p),q);
 +
            }
 +
 
 +
            void l(string s, pair p, pair d) {
 +
                label(s, t(p),d);
 +
            }
 +
           
 +
            d(A--B--C--cycle);
 +
            d(A--D);
 +
            d(B--E);
 +
            o(A);
 +
            o(B);
 +
            o(C);
 +
            o(D);
 +
            o(E);
 +
            o(G);
 +
            l("$A$",A,N);
 +
            l("$B$",B,SW);
 +
            l("$C$",C,SE);
 +
            l("$D$",D,S);
 +
            l("$E$",E,NE);
 +
            l("$G$",G,NW);
 +
</asy>
 +
 
 +
==Solution 1 (Law of Cosines)==
 +
 
 +
Let <math>AG=AE=EG=2x</math>. Since <math>E</math> is the midpoint of <math>\overline{AC}</math>, we must have <math>EC=2x</math>.
 +
 
 +
Since the centroid splits the median in a <math>2:1</math> ratio, <math>GD=x</math> and <math>BG=4x</math>.
 +
 
 +
Applying Law of Cosines on <math>\triangle ADC</math> and <math>\triangle{}AGB</math> yields <math>AB=\sqrt{28}x</math> and <math>CD=BD=\sqrt{13}x</math>. Finally, applying Law of Cosines on <math>\triangle ABC</math> yields <math>\cos(C)=\frac{5}{2\sqrt{13}}=\frac{5\sqrt{13}}{26}</math>. The requested sum is <math>5+13+26=44</math>.
 +
 
 +
==Solution 2 (Law of Cosines: One Fewer Step) ==
 +
 
 +
Let <math>AG = 1</math>. Since <math>\frac{BG}{GE}=2</math> (as <math>G</math> is the centroid), <math>BE = 3</math>. Also, <math>EC = 1</math> and <math>\angle{BEC} = 120^{\circ}</math>. By the law of cosines (applied on <math>\triangle BEC</math>), <math>BC = \sqrt{13}</math>.
 +
 
 +
Applying the law of cosines again on <math>\triangle BEC</math> gives <math>\cos{\angle{C}} = \frac{1 + 13 - 9}{2\sqrt{13}} = \frac{5\sqrt{13}}{26}</math>, so the answer is <math>\fbox{\textbf{(A)}\ 44}</math>.
 +
 
 +
~[[User:Bxiao31415 | Bxiao31415]]
 +
 
 +
==Solution 3 (Law of Cosine)==
 +
 
 +
Let <math>AG = AE = GE = CE = 1</math>. Since <math>G</math> is the centroid, <math>DG = \frac12</math>, <math>BG = 2</math>.
 +
 
 +
<cmath>\angle BGD = \angle AGE = 60^{\circ}</cmath>
 +
 
 +
By the Law of Cosine in <math>\triangle BGD</math>
 +
 
 +
<cmath>BD^2 = BG^2 + DG^2 - 2 \cdot BG \cdot DG \cdot \cos \angle BGD</cmath>
 +
 
 +
<cmath>BD = \sqrt {2^2 + \left( \frac{1}{2} \right)^2 - 2 \cdot 2 \cdot \frac12 \cdot \cos \angle BGD} = \frac{\sqrt{13}}{2}, \quad CD = \frac{\sqrt{13}}{2}</cmath>
 +
 
 +
By the Law of Cosine in <math>\triangle ACD</math>
 +
 
 +
<cmath>AD^2 = AC^2 + CD^2 - 2 \cdot AC \cdot CD \cdot \cos \angle C</cmath>
 +
 
 +
<cmath>\cos \angle C = \frac{ AC^2 + CD^2 - AD^2 }{ 2 \cdot AC \cdot CD } = \frac{ 2^2 + \left( \frac{\sqrt{13}}{2} \right)^2 - \left( \frac{3}{2} \right)^2 }{ 2 \cdot 2 \cdot \frac{\sqrt{13}}{2} } = \frac{ 5 \sqrt{13} }{26}</cmath>
 +
 
 +
<cmath> 5 + 13 + 26 = \boxed{\textbf{(A) }44}</cmath>
 +
 
 +
~[https://artofproblemsolving.com/wiki/index.php/User:Isabelchen isabelchen]
 +
 
 +
==Solution 4 (Barycentric Coordinates)==
 +
Using reference triangle <math>\triangle AGE</math>, we can let <cmath>A=(1,0,0),G=(0,1,0),E=(0,0,1),C=(-1,0,2),D=(-\tfrac{1}{2},\tfrac{3}{2},0),B=(0,3,-2).</cmath> If we move <math>A,B,C</math> each over by <math>(1,0,-2)</math>, leaving <math>\angle C</math> unchanged, we have <cmath>A=(2,0,-2),B=(1,3,-4),C=(0,0,0).</cmath> The angle <math>\theta</math> between vectors <math>\overrightarrow{CA}</math> and <math>\overrightarrow{CB}</math> satisfies <cmath>\cos\theta=\frac{(2)(1)+(0)(3)+(-2)(-4)}{\sqrt{\left[2^{2}+0^{2}+(-2)^{2}\right]\left[1^{2}+3^{2}+(-4)^{2}\right]}}=\frac{10}{\sqrt{8\cdot 26}}=\frac{10}{4\sqrt{13}}=\frac{5\sqrt{13}}{26},</cmath> giving the answer, <math>5+13+26=\boxed{\textbf{(A)}~44}</math>.
 +
 
 +
~r00tsOfUnity
 +
 
 +
==Video Solution by MOP 2024==
 +
https://youtu.be/QNjvpYI1V5g
 +
 
 +
~r00tsOfUnity
 +
 
 +
==Video Solution (Just 3 min!)==
 +
https://youtu.be/Q54sH65AJa4
 +
 
 +
<i>~Education, the Study of Everything</i>
 +
 
 +
==Video Solution(Length & Angle Chasing)==
 +
https://youtu.be/JVDlHCSPF6k
 +
 
 +
~Hayabusa1
  
 
== See Also ==
 
== See Also ==
 
{{AMC12 box|year=2022|ab=B|num-b=18|num-a=20}}
 
{{AMC12 box|year=2022|ab=B|num-b=18|num-a=20}}
 
{{MAA Notice}}
 
{{MAA Notice}}

Latest revision as of 03:53, 12 November 2023

Problem

In $\triangle{ABC}$ medians $\overline{AD}$ and $\overline{BE}$ intersect at $G$ and $\triangle{AGE}$ is equilateral. Then $\cos(C)$ can be written as $\frac{m\sqrt p}n$, where $m$ and $n$ are relatively prime positive integers and $p$ is a positive integer not divisible by the square of any prime. What is $m+n+p?$

$\textbf{(A) }44 \qquad \textbf{(B) }48 \qquad \textbf{(C) }52 \qquad \textbf{(D) }56 \qquad \textbf{(E) }60$

Diagram

[asy]             import geometry;             unitsize(2cm);  			real arg(pair p) {               return atan2(p.y, p.x) * 180/pi;             }              pair G=(0,0),E=(1,0),A=(1/2,sqrt(3)/2),D=1.5*G-0.5*A,C=2*E-A,B=2*D-C;              pair t(pair p) {                 return rotate(-arg(dir(B--C)))*p;             }               path t(path p) {                 return rotate(-arg(dir(B--C)))*p;             }              void d(path p, pen q = black+linewidth(1.5)) {                 draw(t(p),q);             }              void o(pair p, pen q = 5+black) {                 dot(t(p),q);             }              void l(string s, pair p, pair d) {                 label(s, t(p),d);             }                          d(A--B--C--cycle);             d(A--D);             d(B--E);             o(A);             o(B);             o(C);             o(D);             o(E);             o(G);             l("$A$",A,N);             l("$B$",B,SW);             l("$C$",C,SE);             l("$D$",D,S);             l("$E$",E,NE);             l("$G$",G,NW); [/asy]

Solution 1 (Law of Cosines)

Let $AG=AE=EG=2x$. Since $E$ is the midpoint of $\overline{AC}$, we must have $EC=2x$.

Since the centroid splits the median in a $2:1$ ratio, $GD=x$ and $BG=4x$.

Applying Law of Cosines on $\triangle ADC$ and $\triangle{}AGB$ yields $AB=\sqrt{28}x$ and $CD=BD=\sqrt{13}x$. Finally, applying Law of Cosines on $\triangle ABC$ yields $\cos(C)=\frac{5}{2\sqrt{13}}=\frac{5\sqrt{13}}{26}$. The requested sum is $5+13+26=44$.

Solution 2 (Law of Cosines: One Fewer Step)

Let $AG = 1$. Since $\frac{BG}{GE}=2$ (as $G$ is the centroid), $BE = 3$. Also, $EC = 1$ and $\angle{BEC} = 120^{\circ}$. By the law of cosines (applied on $\triangle BEC$), $BC = \sqrt{13}$.

Applying the law of cosines again on $\triangle BEC$ gives $\cos{\angle{C}} = \frac{1 + 13 - 9}{2\sqrt{13}} = \frac{5\sqrt{13}}{26}$, so the answer is $\fbox{\textbf{(A)}\ 44}$.

~ Bxiao31415

Solution 3 (Law of Cosine)

Let $AG = AE = GE = CE = 1$. Since $G$ is the centroid, $DG = \frac12$, $BG = 2$.

\[\angle BGD = \angle AGE = 60^{\circ}\]

By the Law of Cosine in $\triangle BGD$

\[BD^2 = BG^2 + DG^2 - 2 \cdot BG \cdot DG \cdot \cos \angle BGD\]

\[BD = \sqrt {2^2 + \left( \frac{1}{2} \right)^2 - 2 \cdot 2 \cdot \frac12 \cdot \cos \angle BGD} = \frac{\sqrt{13}}{2}, \quad CD = \frac{\sqrt{13}}{2}\]

By the Law of Cosine in $\triangle ACD$

\[AD^2 = AC^2 + CD^2 - 2 \cdot AC \cdot CD \cdot \cos \angle C\]

\[\cos \angle C = \frac{ AC^2 + CD^2 - AD^2 }{ 2 \cdot AC \cdot CD } = \frac{ 2^2 + \left( \frac{\sqrt{13}}{2} \right)^2 - \left( \frac{3}{2} \right)^2 }{ 2 \cdot 2 \cdot \frac{\sqrt{13}}{2} } = \frac{ 5 \sqrt{13} }{26}\]

\[5 + 13 + 26 = \boxed{\textbf{(A) }44}\]

~isabelchen

Solution 4 (Barycentric Coordinates)

Using reference triangle $\triangle AGE$, we can let \[A=(1,0,0),G=(0,1,0),E=(0,0,1),C=(-1,0,2),D=(-\tfrac{1}{2},\tfrac{3}{2},0),B=(0,3,-2).\] If we move $A,B,C$ each over by $(1,0,-2)$, leaving $\angle C$ unchanged, we have \[A=(2,0,-2),B=(1,3,-4),C=(0,0,0).\] The angle $\theta$ between vectors $\overrightarrow{CA}$ and $\overrightarrow{CB}$ satisfies \[\cos\theta=\frac{(2)(1)+(0)(3)+(-2)(-4)}{\sqrt{\left[2^{2}+0^{2}+(-2)^{2}\right]\left[1^{2}+3^{2}+(-4)^{2}\right]}}=\frac{10}{\sqrt{8\cdot 26}}=\frac{10}{4\sqrt{13}}=\frac{5\sqrt{13}}{26},\] giving the answer, $5+13+26=\boxed{\textbf{(A)}~44}$.

~r00tsOfUnity

Video Solution by MOP 2024

https://youtu.be/QNjvpYI1V5g

~r00tsOfUnity

Video Solution (Just 3 min!)

https://youtu.be/Q54sH65AJa4

~Education, the Study of Everything

Video Solution(Length & Angle Chasing)

https://youtu.be/JVDlHCSPF6k

~Hayabusa1

See Also

2022 AMC 12B (ProblemsAnswer KeyResources)
Preceded by
Problem 18
Followed by
Problem 20
1 2 3 4 5 6 7 8 9 10 11 12 13 14 15 16 17 18 19 20 21 22 23 24 25
All AMC 12 Problems and Solutions

The problems on this page are copyrighted by the Mathematical Association of America's American Mathematics Competitions. AMC logo.png